How Do You Derive the Vector Identity Involving Divergence and Curl?

Click For Summary
The discussion focuses on deriving the vector identity involving divergence and curl, specifically the equation ∇(F . G) = (F . ∇)G + (G . ∇)F + F x (∇ x G) + G x (∇ x F). The initial approach using the BAC-CAB rule leads to confusion, as it does not apply correctly due to the nature of the del operator compared to standard vectors. The participants clarify that the commutation of operators introduces additional terms, which must be accounted for. By applying partial operators and treating components separately, the correct derivation of the identity is achieved. Understanding the distinction between vector cross products and operator manipulations is crucial for resolving the identity correctly.
bcjochim07
Messages
366
Reaction score
0

Homework Statement


The vectors F and G are arbitrary functions of position. Starting w/ the relations F x (∇ x G) and G x (∇ x F), obtain the identity

∇(F . G) = (F . ∇)G + (G . ∇)F + F x (∇ x G) + G x (∇ x F)


Homework Equations





The Attempt at a Solution



I started off with the relation F x (∇ x G) and used the BAC-CAB rule:

F x (∇ x G) = ∇(F . G) - (G . ∇)F

so ∇(F . G = (G . ∇)F + F x (∇ x G) which seems to contradict the identity I am supposed to get. What am I doing wrong?
 
Physics news on Phys.org
you can't apply the BAC-CAB rule here. That was derived for vectors by assuming that A X B = - B X A

However with the del operator del X A is a vector, but A X del is an operator...so there's a world of difference
 
The trouble is commuting an opperator adds a commutator term
In single variable calculus
D(uv)=uDv+vDu not uDv
we can use partial opperators to avoid this
let a opperant in {} be fixed
D(uv)=D({u}v)+D(u{v})={u}Dv+{v}Du=uDv+vDu

∇(F . G )=∇({F} . G )+∇(F . {G} )
∇({F} . G )=Fx(∇xG)+(F.∇)G
∇(F . {G} )=Gx(∇xF)+(G.∇)F
∇(F . G )=∇({F} . G )+∇(F . {G} )=Fx(∇xG)+(F.∇)G+Gx(∇xF)+(G.∇)F
 
Question: A clock's minute hand has length 4 and its hour hand has length 3. What is the distance between the tips at the moment when it is increasing most rapidly?(Putnam Exam Question) Answer: Making assumption that both the hands moves at constant angular velocities, the answer is ## \sqrt{7} .## But don't you think this assumption is somewhat doubtful and wrong?

Similar threads

Replies
3
Views
1K
  • · Replies 1 ·
Replies
1
Views
2K
  • · Replies 4 ·
Replies
4
Views
2K
  • · Replies 2 ·
Replies
2
Views
2K
  • · Replies 6 ·
Replies
6
Views
2K
  • · Replies 8 ·
Replies
8
Views
2K
  • · Replies 4 ·
Replies
4
Views
2K
  • · Replies 2 ·
Replies
2
Views
1K
Replies
2
Views
2K
Replies
1
Views
1K